Indukcja matematyczna.

Teoria liczb, teoria grafów, indukcja
Otrzymałeś(aś) rozwiązanie do zamieszczonego zadania? - podziękuj autorowi rozwiązania! Kliknij
qweasdzxc123321
Dopiero zaczynam
Dopiero zaczynam
Posty: 21
Rejestracja: 02 maja 2015, 19:07
Podziękowania: 16 razy
Płeć:

Indukcja matematyczna.

Post autor: qweasdzxc123321 »

\(( \forall n \in N, n \ge 2) \sum_{i=1}^{n} \frac{1}{ \sqrt{i} }>2( \sqrt{n+1}-1)\)
qweasdzxc123321
Dopiero zaczynam
Dopiero zaczynam
Posty: 21
Rejestracja: 02 maja 2015, 19:07
Podziękowania: 16 razy
Płeć:

Re: Indukcja matematyczna.

Post autor: qweasdzxc123321 »

To jest az tak trudne czy nikt "odpowiedni" jeszcze tego nie widzial? :P
radagast
Guru
Guru
Posty: 17549
Rejestracja: 09 lis 2010, 07:38
Lokalizacja: Warszawa
Podziękowania: 41 razy
Otrzymane podziękowania: 7435 razy
Płeć:

Re: Indukcja matematyczna.

Post autor: radagast »

dla \(n=2\) mamy: \(1+ \frac{1}{ \sqrt{2} } >2( \sqrt{3} -1)\) bo \(1+ \frac{1}{ \sqrt{2} } >2 \sqrt{3} -2\) bo \(3+ \frac{1}{ \sqrt{2} } >2 \sqrt{3}> 3\) OK
zał ind : istnieje \(n \in N\) t. że \(\displaystyle \sum_{i=1}^{n} \frac{1}{ \sqrt{i} }>2( \sqrt{n+1}-1)\)
teza \(\displaystyle \sum_{i=1}^{n+1} \frac{1}{ \sqrt{i} }>2( \sqrt{n+2}-1)\)
DOWÓD
\(\displaystyle L=\sum_{i=1}^{n+1} \frac{1}{ \sqrt{i} }=\sum_{i=1}^{n} \frac{1}{ \sqrt{i} }+ \frac{1}{ \sqrt{n+1} }>2( \sqrt{n+1}-1)+ \frac{1}{ \sqrt{n+1} }=2 \sqrt{n+1}+ \frac{1}{ \sqrt{n+1} } -2=\\

\displaystyle \frac{2 (n+1)}{ \sqrt{n+1}} + \frac{1}{ \sqrt{n+1} } -2= \frac{2 n+3}{ \sqrt{n+1}} -2=\frac{ \sqrt{4n^2+12n+9} }{ \sqrt{n+1}} -2>\frac{ \sqrt{4n^2+12n+8} }{ \sqrt{n+1}} -2=\\

\displaystyle \frac{ \sqrt{4(n+1)(n+2)} }{ \sqrt{n+1}} -2=2( \sqrt{n+2}-1)=P\ \ \ \ \So \ \ \ L>P\)

c.b.d.o.
qweasdzxc123321
Dopiero zaczynam
Dopiero zaczynam
Posty: 21
Rejestracja: 02 maja 2015, 19:07
Podziękowania: 16 razy
Płeć:

Re: Indukcja matematyczna.

Post autor: qweasdzxc123321 »

Dziękuje :)
ODPOWIEDZ